Legend of Minerva
Verifiziertes Spiel


- Platform: Android
- Aktualisiert: 25.01.2025
- Android Spielversion: 5.0
-
Language:
- Aktuelle Version: 2021.6.30
- Google Play: -
Verwandeln Sie sich in einen wahren Ritter und besiegen Sie böswillige Mächte im Fantasy-Online-Rollenspiel Legend of Minerva.
Highlights: Erkunden Sie über 1000 lebendige Karten, laden Sie Freunde zu Boni ein, nehmen Sie an epischen Kämpfen gegen Bosse teil, gründen Sie Gilden und tauchen Sie ein in eine außergewöhnliche Handlung.<|endoftext|><|endoftext|>
# 2015 AMC 10B Probleme/Problem 1.
## Inhalt.
1 Problem
2 Lösung
3 Videolösung
4 Siehe auch
## Problem
Was ist der Wert von 2+4+6+8+10+12+14+16+18+20$?
$\textbf{(A)}\ 90\qquad\textbf{(B)}\ 100\qquad\textbf{(C)}\ 110\qquad\textbf{(D)}\ 120\qquad\textbf{(E )}\ 130$
## Lösung.
Wir können die Zahlen wie folgt paaren: $(2+20)+(4+18)+(6+16)+(8+14)+(10+12)$. Jedes Paar ergibt zusammen $22$, und es gibt $5$-Paare, also ist die Summe $22 \cdot 5 = \boxed{\textbf{(B)}\ 100}$.
## Videolösung.
https://youtu.be/8WrdYLw9_ns
~ Savannahsolver
<|endoftext|>## Mathematische Foren
## Kategorie: Oberstufenolympiaden
## Thema: Ungleichheit
## Aufrufe: 338
## [Eingabe: math-user1, num_posts=697, num_likes_received=372]
## [math-user1, num_likes=1]
Seien $a,b,c$ positive reelle Zahlen, so dass $a+b+c=3$. Beweisen Sie das
$\frac{a}{b}+\frac{b}{c}+\frac{c}{a}+3\geq 2(ab+bc+ca)$
## [Eingabe: math-user2, num_posts=467, num_likes_received=180]
## [math-user2, num_likes=0]
Nach AM-GM ist $ab+bc+ca\le\frac{\left(a+b+c\right)^2}{3}=3$. Wir müssen also $\frac{a}{b}+\frac{b}{c}+\frac{c}{a}\ge3$ beweisen. Von AM-GM, $\frac{a}{b}+\frac{a}{b}+\frac{b}{c}\ge3\sqrt[3]{\frac{a^2}{bc}}=3\sqrt [3]{\frac{ a^3}{abc}}=3\sqrt[3]{\frac{a^3}{abc}\cdot\frac{b^3}{abc}\cdot\frac{c^3}{abc} }=3$. Durch zyklisches Summieren erhält man das gewünschte Ergebnis.
## [Eingabe: math-user3, num_posts=545, num
